Question and Answers Forum

All Questions      Topic List

Algebra Questions

Previous in All Question      Next in All Question      

Previous in Algebra      Next in Algebra      

Question Number 57521 by rahul 19 last updated on 06/Apr/19

Commented by rahul 19 last updated on 06/Apr/19

Ans−(a)11.

$${Ans}−\left({a}\right)\mathrm{11}. \\ $$

Answered by tanmay.chaudhury50@gmail.com last updated on 07/Apr/19

T_n =x^2 +nx+(n−1)x+(n^2 −n)  T_n =x^2 +(2n−1)x+(n^2 −n)  T_1 =x^2 +(2.1−1)x+(1^2 −1)  T_2 =x^2 +(2.2−1)x+(2^2 −2)  T_3 =x^2 +(2.3−1)x+(3^2 −3)  ...  ...  on addition  ΣT_n =nx^2 +[2.(n/2){2.1+(n−1)1}−n]x+((n(n+1)(2n+1))/6)−((n(n+1))/2)  =nx^2 +[n(n+1)−n]x+((n(n+1))/2)(((2n+1)/3)−1)  =nx^2 +(n^2 x)+((n(n+1))/2)(((2n−2)/3))  =nx^2 +n^2 x+(n/3)(n^2 −1)  so the eqn is  nx^2 +n^2 x+(n/3)(n^2 −1)=10n  x^2 +nx+((n^2 −1)/3)−10=0  given β=α+1  (α−β)^2 =(α+β)^2 −4αβ  1=(−n)^2 −4[((n^2 −1)/3)−10]  1=n^2 −4(((n^2 −1−30)/3))  3=3n^2 −4n^2 +124  n^2 =121  n=11

$${T}_{{n}} ={x}^{\mathrm{2}} +{nx}+\left({n}−\mathrm{1}\right){x}+\left({n}^{\mathrm{2}} −{n}\right) \\ $$$${T}_{{n}} ={x}^{\mathrm{2}} +\left(\mathrm{2}{n}−\mathrm{1}\right){x}+\left({n}^{\mathrm{2}} −{n}\right) \\ $$$${T}_{\mathrm{1}} ={x}^{\mathrm{2}} +\left(\mathrm{2}.\mathrm{1}−\mathrm{1}\right){x}+\left(\mathrm{1}^{\mathrm{2}} −\mathrm{1}\right) \\ $$$${T}_{\mathrm{2}} ={x}^{\mathrm{2}} +\left(\mathrm{2}.\mathrm{2}−\mathrm{1}\right){x}+\left(\mathrm{2}^{\mathrm{2}} −\mathrm{2}\right) \\ $$$${T}_{\mathrm{3}} ={x}^{\mathrm{2}} +\left(\mathrm{2}.\mathrm{3}−\mathrm{1}\right){x}+\left(\mathrm{3}^{\mathrm{2}} −\mathrm{3}\right) \\ $$$$... \\ $$$$... \\ $$$${on}\:{addition} \\ $$$$\Sigma{T}_{{n}} ={nx}^{\mathrm{2}} +\left[\mathrm{2}.\frac{{n}}{\mathrm{2}}\left\{\mathrm{2}.\mathrm{1}+\left({n}−\mathrm{1}\right)\mathrm{1}\right\}−{n}\right]{x}+\frac{{n}\left({n}+\mathrm{1}\right)\left(\mathrm{2}{n}+\mathrm{1}\right)}{\mathrm{6}}−\frac{{n}\left({n}+\mathrm{1}\right)}{\mathrm{2}} \\ $$$$={nx}^{\mathrm{2}} +\left[{n}\left({n}+\mathrm{1}\right)−{n}\right]{x}+\frac{{n}\left({n}+\mathrm{1}\right)}{\mathrm{2}}\left(\frac{\mathrm{2}{n}+\mathrm{1}}{\mathrm{3}}−\mathrm{1}\right) \\ $$$$={nx}^{\mathrm{2}} +\left({n}^{\mathrm{2}} {x}\right)+\frac{{n}\left({n}+\mathrm{1}\right)}{\mathrm{2}}\left(\frac{\mathrm{2}{n}−\mathrm{2}}{\mathrm{3}}\right) \\ $$$$={nx}^{\mathrm{2}} +{n}^{\mathrm{2}} {x}+\frac{{n}}{\mathrm{3}}\left({n}^{\mathrm{2}} −\mathrm{1}\right) \\ $$$${so}\:{the}\:{eqn}\:{is} \\ $$$${nx}^{\mathrm{2}} +{n}^{\mathrm{2}} {x}+\frac{{n}}{\mathrm{3}}\left({n}^{\mathrm{2}} −\mathrm{1}\right)=\mathrm{10}{n} \\ $$$${x}^{\mathrm{2}} +{nx}+\frac{{n}^{\mathrm{2}} −\mathrm{1}}{\mathrm{3}}−\mathrm{10}=\mathrm{0} \\ $$$${given}\:\beta=\alpha+\mathrm{1} \\ $$$$\left(\alpha−\beta\right)^{\mathrm{2}} =\left(\alpha+\beta\right)^{\mathrm{2}} −\mathrm{4}\alpha\beta \\ $$$$\mathrm{1}=\left(−{n}\right)^{\mathrm{2}} −\mathrm{4}\left[\frac{{n}^{\mathrm{2}} −\mathrm{1}}{\mathrm{3}}−\mathrm{10}\right] \\ $$$$\mathrm{1}={n}^{\mathrm{2}} −\mathrm{4}\left(\frac{{n}^{\mathrm{2}} −\mathrm{1}−\mathrm{30}}{\mathrm{3}}\right) \\ $$$$\mathrm{3}=\mathrm{3}{n}^{\mathrm{2}} −\mathrm{4}{n}^{\mathrm{2}} +\mathrm{124} \\ $$$${n}^{\mathrm{2}} =\mathrm{121} \\ $$$${n}=\mathrm{11} \\ $$$$ \\ $$$$ \\ $$

Commented by rahul 19 last updated on 07/Apr/19

thanks sir!

$${thanks}\:{sir}! \\ $$

Commented by peter frank last updated on 07/Apr/19

thank you

$${thank}\:{you} \\ $$

Answered by Rasheed.Sindhi last updated on 07/Apr/19

x(x+1)+(x+1)(x+2)+...(x+n−1^(−) )(x+n)=10n  nx^2 +n^2 x+(n/3)(n^2 −1)=10n                [ From tanmay sir]  Let α & (α+1) are solutions:  α(α+1)+(α+1)(α+2)+...(α+n−1^(−) )(α+n)=10n...(I)  (α+1)(α+2)+(α+2)(α+3)...(α+n)(α+n+1^(−) )=10n...(II)  (I)−(II):  α(α+1)−(α+n)(α+n+1)=0  α^2 +α−[α^2 +(2n+1)α+n(n+1)]=0  −2nα−n^2 −n=0  n^2 +(2α+1)n=0  n(n+2α+1)=0  As  n>0 ⇒n+2α+1=0      n=−2α−1  According the guidance of sir mr W:  α=−((n+1)/2)  Putting in (I)  (-((n+1)/2))(-((n+1)/2)+1)+(-((n+1)/2)+1)(-((n+1)/2)+2)+...(-((n+1)/2)+n−1^(−) )(-((n+1)/2)+n)=10n...(I)  Or equivalently according to sir tanmay    n(-((n+1)/2))^2 +n^2 (-((n+1)/2))+(n/3)(n^2 −1)=10n  ((n(n+1)^2 )/4)−((n^2 (n+1))/2)+((n(n^2 −1))/3)=10n  3n(n+1)^2 −6n^2 (n+1)+4n(n^2 −1)=120n  3n^3 +6n^2 +3n−6n^3 −6n^2 +4n^3 −4n−120n=0  n^3 −121n=0  n(n^2 −121)=0  ∵ n>0⇒n^2 =121⇒n=11

$${x}\left({x}+\mathrm{1}\right)+\left({x}+\mathrm{1}\right)\left({x}+\mathrm{2}\right)+...\left({x}+\overline {{n}−\mathrm{1}}\right)\left({x}+{n}\right)=\mathrm{10}{n} \\ $$$${nx}^{\mathrm{2}} +{n}^{\mathrm{2}} {x}+\frac{{n}}{\mathrm{3}}\left({n}^{\mathrm{2}} −\mathrm{1}\right)=\mathrm{10}{n} \\ $$$$\:\:\:\:\:\:\:\:\:\:\:\:\:\:\left[\:{From}\:{tanmay}\:{sir}\right] \\ $$$${Let}\:\alpha\:\&\:\left(\alpha+\mathrm{1}\right)\:{are}\:{solutions}: \\ $$$$\alpha\left(\alpha+\mathrm{1}\right)+\left(\alpha+\mathrm{1}\right)\left(\alpha+\mathrm{2}\right)+...\left(\alpha+\overline {{n}−\mathrm{1}}\right)\left(\alpha+{n}\right)=\mathrm{10}{n}...\left({I}\right) \\ $$$$\left(\alpha+\mathrm{1}\right)\left(\alpha+\mathrm{2}\right)+\left(\alpha+\mathrm{2}\right)\left(\alpha+\mathrm{3}\right)...\left(\alpha+{n}\right)\left(\alpha+\overline {{n}+\mathrm{1}}\right)=\mathrm{10}{n}...\left({II}\right) \\ $$$$\left({I}\right)−\left({II}\right): \\ $$$$\alpha\left(\alpha+\mathrm{1}\right)−\left(\alpha+{n}\right)\left(\alpha+{n}+\mathrm{1}\right)=\mathrm{0} \\ $$$$\alpha^{\mathrm{2}} +\alpha−\left[\alpha^{\mathrm{2}} +\left(\mathrm{2}{n}+\mathrm{1}\right)\alpha+{n}\left({n}+\mathrm{1}\right)\right]=\mathrm{0} \\ $$$$−\mathrm{2}{n}\alpha−{n}^{\mathrm{2}} −{n}=\mathrm{0} \\ $$$${n}^{\mathrm{2}} +\left(\mathrm{2}\alpha+\mathrm{1}\right){n}=\mathrm{0} \\ $$$${n}\left({n}+\mathrm{2}\alpha+\mathrm{1}\right)=\mathrm{0} \\ $$$${As}\:\:{n}>\mathrm{0}\:\Rightarrow{n}+\mathrm{2}\alpha+\mathrm{1}=\mathrm{0} \\ $$$$\:\:\:\:{n}=−\mathrm{2}\alpha−\mathrm{1} \\ $$$${According}\:{the}\:{guidance}\:{of}\:{sir}\:{mr}\:{W}: \\ $$$$\alpha=−\frac{{n}+\mathrm{1}}{\mathrm{2}} \\ $$$${Putting}\:{in}\:\left({I}\right) \\ $$$$\left(-\frac{{n}+\mathrm{1}}{\mathrm{2}}\right)\left(-\frac{{n}+\mathrm{1}}{\mathrm{2}}+\mathrm{1}\right)+\left(-\frac{{n}+\mathrm{1}}{\mathrm{2}}+\mathrm{1}\right)\left(-\frac{{n}+\mathrm{1}}{\mathrm{2}}+\mathrm{2}\right)+...\left(-\frac{{n}+\mathrm{1}}{\mathrm{2}}+\overline {{n}−\mathrm{1}}\right)\left(-\frac{{n}+\mathrm{1}}{\mathrm{2}}+{n}\right)=\mathrm{10}{n}...\left({I}\right) \\ $$$${Or}\:{equivalently}\:{according}\:{to}\:{sir}\:{tanmay}\: \\ $$$$\:{n}\left(-\frac{{n}+\mathrm{1}}{\mathrm{2}}\right)^{\mathrm{2}} +{n}^{\mathrm{2}} \left(-\frac{{n}+\mathrm{1}}{\mathrm{2}}\right)+\frac{{n}}{\mathrm{3}}\left({n}^{\mathrm{2}} −\mathrm{1}\right)=\mathrm{10}{n} \\ $$$$\frac{{n}\left({n}+\mathrm{1}\right)^{\mathrm{2}} }{\mathrm{4}}−\frac{{n}^{\mathrm{2}} \left({n}+\mathrm{1}\right)}{\mathrm{2}}+\frac{{n}\left({n}^{\mathrm{2}} −\mathrm{1}\right)}{\mathrm{3}}=\mathrm{10}{n} \\ $$$$\mathrm{3}{n}\left({n}+\mathrm{1}\right)^{\mathrm{2}} −\mathrm{6}{n}^{\mathrm{2}} \left({n}+\mathrm{1}\right)+\mathrm{4}{n}\left({n}^{\mathrm{2}} −\mathrm{1}\right)=\mathrm{120}{n} \\ $$$$\mathrm{3}{n}^{\mathrm{3}} +\mathrm{6}{n}^{\mathrm{2}} +\mathrm{3}{n}−\mathrm{6}{n}^{\mathrm{3}} −\mathrm{6}{n}^{\mathrm{2}} +\mathrm{4}{n}^{\mathrm{3}} −\mathrm{4}{n}−\mathrm{120}{n}=\mathrm{0} \\ $$$${n}^{\mathrm{3}} −\mathrm{121}{n}=\mathrm{0} \\ $$$${n}\left({n}^{\mathrm{2}} −\mathrm{121}\right)=\mathrm{0} \\ $$$$\because\:{n}>\mathrm{0}\Rightarrow{n}^{\mathrm{2}} =\mathrm{121}\Rightarrow{n}=\mathrm{11} \\ $$

Commented by mr W last updated on 07/Apr/19

you are not wrong sir. you have two  unknowns α and n, and you have two  equations I and II. now you have got  n=−2α−1 or α=−((n+1)/2), you can put  this into I to find n.

$${you}\:{are}\:{not}\:{wrong}\:{sir}.\:{you}\:{have}\:{two} \\ $$$${unknowns}\:\alpha\:{and}\:{n},\:{and}\:{you}\:{have}\:{two} \\ $$$${equations}\:{I}\:{and}\:{II}.\:{now}\:{you}\:{have}\:{got} \\ $$$${n}=−\mathrm{2}\alpha−\mathrm{1}\:{or}\:\alpha=−\frac{{n}+\mathrm{1}}{\mathrm{2}},\:{you}\:{can}\:{put} \\ $$$${this}\:{into}\:{I}\:{to}\:{find}\:{n}. \\ $$

Commented by Rasheed.Sindhi last updated on 07/Apr/19

θNX   S iR mr W!

$$\theta{NX}\:\:\:{S}\:{i}\mathcal{R}\:{mr}\:{W}! \\ $$

Terms of Service

Privacy Policy

Contact: info@tinkutara.com